give the distance between two points with the given coordinates.
-10.3 and -12.1

Give The Distance Between Two Points With The Given Coordinates. -10.3 And -12.1

Answers

Answer 1
I believe the answer is 2.8

Related Questions

You go to the grocery store to buy milk and some eggs. Milk costs $3.67 and eggs cost $2.40. You want to make sure you have enough money. Estimate the cost of these two items to the nearest ones place.

Answers

Answer:

Step-by-step explanation:

Let's start by round 3.67. We look at the key digit which is 6 in this case. Since 6 is greater than 5, 3.67 will round up to 4.

The key digit in 2.40 is 4. 4 is not greater than 5, so we round down to 2.

2+4=6

Therefore, an estimate of these two items to the nearest ones place is $6.

Answer:

$2 for the eggs and $4 for the milk

Step-by-step explanation:

Since the eggs cost $2.40 and after the 2 you can see that there is a 4, " 5 or more we add one more, 4 or less just ignore", so it stays $2. However the milk costs $3.67, and here you can see that after the 3 there is a 4, so since it's more than 5 we change the estimated amount to $4.

I hope you understood, and I hope the answer is right :)

Is 9pi rational or irrational

Answers

Answer:

irrational

Step-by-step explanation:

every number that times with pi are irrational, only zero gives a rational number

Find the output, k, when the input, t, is -7.

k= 10t - 19

k=

Answers

Answer:

Here is the answer . hope this helps.

The area of this shape is 24 square units. Which of these statements is true about the area ?​

Answers

D.

Because the area of an object is number of square units or square grids inside the shape.

Answer:

D.

Step-by-step explanation:

Lindsay prepared 9 containers of fruit punch for a party. After the party, there were still 4 4/9 containers of fruit punch left. How many containers were consumed in the party?

Answers

Turn 9 into [tex]\frac{81}{9}[/tex]. Then, convert 4 [tex]\frac{4}{9}[/tex] into a improper faction ( [tex]\frac{40}{9}[/tex] ). Then, subtract

[tex]\frac{81}{9} - \frac{40}{9} = \frac{41}{9}[/tex]. Lastly, convert  [tex]\frac{41}{9}[/tex] to a mixed fraction ( [tex]4\frac{4}{9}[/tex] ).

Your answer is [tex]4\frac{4}{9}[/tex].

Which statement best describes the relationship between the two figures?

Answers

Answer:

The congruency of triangle is the statement that describes the relationship between two figures Step-by-step explanation:

Assume that an investment of $3000 earns an APR of 6% compounded monthly for 18 months.

How much money is in your account after 18 months?

Answers

9514 1404 393

Answer:

  $3281.79

Step-by-step explanation:

The compound interest formula is ...

  A = P(1 +r/n)^(nt)

where A is the balance resulting from investment of P at annual rate r compounded n times per year for t years.

We have P=$3000, r=0.06, n=12, t=1.5, so the account balance is ...

  A = $3000(1 +0.06/12)^(12·1.5) = $3000(1.005^18) ≈ $3281.79

There will be $3281.79 in the account after 18 months.

Combine Like Terms
4. Which expression is equivalent to -9 + 8y + 12 - (-2y) + 1 ?
14y
10y
4+6y
10y+4

Answers

Answer:

10y + 4

Step-by-step explanation:

-9 + 8y + 12 - (-2y) + 1

-9 + 8y + 12 + 2y + 1

8y + 2y - 9 + 12 + 1

10y + 4

5 increased by the product of 2 and t​

Answers

Answer:

Step-by-step explanation:

increased means add.

5 + 2*t

pelase asnwer da question

Answers

Answer:

D

Step-by-step explanation:

I think its D because you multiple the number that is out of the the bracket with the numerators..

so its

a^4/b^12

(1.) Find the equation of the line passing through the points (-4,-6 ) and (-5,5).

(2.) Find the equation of the line with slope -8 that passes through the point (3, -8).

Answers

Answer:

1) y=-11x-50    2)y=-8x+16

Step-by-step explanation:

1.  y=mx+b- it is the main equation for a line'

If we take the first point and use its coordinates we have -6=-4m+b

If we take the second point and use its coordinates we have 5=5m+b

These equations are simultaneous for our line because the line passes through both points

-6=-4m+b

5=-5m+b

If subtract the second one from the first equation

-6-5= -4m+b- (-5m+b)

-11= m

b= -6-44=-50

y=-11x-50

2) y=mx+b

-8=-8*3+b

b=16

y=-8x+16

The inequality - 15<-6 is given. Write the inequality you get when you perform the
following operation.
Divide both sides of the inequality by 3.

Answers

Answer:

-5

Step-by-step explanation:

-15<-6=-15

-15÷3

=-5

Inequalities are used to show unequal expressions. There are five inequalities, and they are: not equal to, greater than, less than, greater than or equal to and less than or equal to.

The inequality when both sides of [tex]-15 < -6[/tex] is divided by 3 is [tex]-5< -2[/tex]

Given that:

[tex]-15 < -6[/tex]

Divide both sides by 3

[tex]-15 \div 3< -6 \div 3[/tex]

Evaluate the division

[tex]-5< -2[/tex]

Hence, the inequality when both sides of [tex]-15 < -6[/tex] is divided by 3 is [tex]-5< -2[/tex]

Read more about inequalities at:

https://brainly.com/question/17194874

Write a two-step equation and solve.

Four more than six times a number is 22

Five less than eleven times a number is 50

Seven less than nine times a number is -16

Answers

Step-by-step explanation:

1) let number=a

six times a number=6a

Condition:

6a+4=22

2) eleven times a number=11a

Condition:

11a-5=50

3) 9 times a number=9a

Condition:

9a-7=-16

Note:if you need to ask any question please let me know.

What is the volume of the right rectangular prism?

a) 21cm^3
b) 42cm^3
c) 120cm^3
d) 240cm^3

Answers

Answer:

c part is correct answer

Answer: D) 240 cm^3

Step-by-step explanation:

On edge

I need help as soon as possible pleaseeeeeee!!!!!!!

Answers

first youd have to get the least common denominator, which would be 6. you do this by multiplying the denominator and numerator by the number needed to get to the least common denominator. this gives you 1 and 2/6 aswell as 3 and 3/6. from there its easier to count from 1 and 2/6 to 3 and 3/6. the final answer would be 2 and 1/6 unit away.

if this helped please give brainliest!

(1/5) ³
evaluate

pls help :)

Answers

0.008

1/5 x 1/5 x 1/5

It will be 1/5 times 1/5 times 1/5
:3

Abigail stays after school for chess club practice. Everyone is in the mood for a snack, so they buy bags of pretzels from the vending machine for $0.95 each. If the club members buy 6 bags of pretzels, how much do they pay in all?

Answers

Answer:

5.70

Step-by-step explanation:

Take the price per bag and multiply by the number of bags

.95 * 6 =5.70

To pay for a 15,300 car, Tony made a down payment of $3900 and took out a loan for the rest. On the loan , he paid monthly payment payment of $252.34 for 4 years.

Answers

Answer:

a. $24,512.32

b. $712.32

Step-by-step explanation:

a. A price of car $15,300, Tony made a down payment of $3900 so car has $11,400 left of a price and took out a loan.

He paid monthly payments of $252.34 for 4 years. Which mean $252.34 is paid for 48 months, multiplication $252.34 and 48 are $12,112.32. So we have monthly payments is $12,112.32

The total amount Tony ended up for the car is $11,400 + $12,112.32 = $24,512.32

b. The interest Tony pay on the loan is $12,112.32 - $11,400 = $712.32

. What is the total cost of 6 books at N8.50k each?​

Answers

Answer:

51

Step-by-step explanation:

1 book= N8.50

therefore 6 books= N8.50 × 6 = N51

Tavon has a gift card for $65 that loses $3 for each 30-day period it is not used. He has another gift card for $55 that loses $2.50 for each 30-day period it is not
used
a. Write and solve an equation for the number of 30-day periods until the value of the gift cards will be equal.
b. What will the value of each card be when they have equal value?
a. If x is the number of 30-day periods, then the equation can be used to find the number of 30-day periods until the values of the gift cards will be equal.
(Type an equation. Use integers or decimals for any numbers in the equation)

Answers

Answer:

a. 65 - 3 x = 55 - 2.5 x

ab. 20

b. 5

Step-by-step explanation:

The equations are y = 110 - 2.5x and y = 90 - 2x. Then the value of each card, when they have equalvalue, will be $10.

What is a linear equation?

The linear equation is given as,

y = mx + c

Where m is the slope of the line and c is the y-intercept of the line.

Let x be the number of days and y be the value of the card.

Tavon has a gift card for $110 that loses $2.50 for each​ 30-day period it is not used. He has another gift card for $90 that loses $2 for each​ 30-day period it is not used. Then the equations will be

y = 110 - 2.5x    ...1

y = 90 - 2x       ...2

From equations 1 and 2, we have

110 - 2.5x = 90 - 2x

20 = 0.5x

x = 40

And the value of y will be

y = 110 - 2.5(40)

y = 110 - 100

y = $10

The equations are y = 110 - 2.5x and y = 90 - 2x. Then the value of each card, when they have equal​ value, will be $10.

More about the linear equation link is given below.

brainly.com/question/11897796

#SPJ2

GH = 4x - 1, and DH = 8. Find x.
Help

Answers

x=197 is the answer for your question

Answer:

x=4.25

------------------------------------------

8+8=4x-1

16=4x-1

4x=16+1

4x=17

x= 17/4

I have no idea if i am correct just a guesstimate

Have a good day

If 18 drinks cost £54, how much will 7 drinks cost ?

Answers

Answer:

£21

Step-by-step explanation:

Cost of 18 drinks = £54

Cost of 1 drink

= Cost of 18 drinks/18

= £54/18

= £3

Cost of 7 drinks

= Cost of 1 drink × 7

= £3 × 7

= £21

1.
.? ESSENTIAL QUESTION
How are the
midpoint and length of a segment on the
coordinate plane determined?

Answers

Answer:

to find midpoint add both"X" coordinates and divide by 2

also add "Y" coordinates and divide by 2

then for line of segment use distance formula which is D=√[(x2-x1)²+(y2-y1)²]

Midpoint is the average of x and y coordinates of the endpoint while the length of a line segment is given by the formula [tex]d = \sqrt {\left( {x_1 - x_2 } \right)^2 + \left( {y_1 - y_2 } \right)^2 }[/tex].

What is a line segment?

A line section that can connect two places is referred to as a segment.

In other words, a line segment is just part of a big line that is straight and going unlimited in both directions.

The line is here! It extends endlessly in both directions and has no beginning or conclusion.

To determine the midpoint of two coordinates (x₁,y₁) and (x₂,y₂ )

Midpoint = [ (x₁ + x₂)/2 + (y₁ + y₂)/2 ]

The distance between two points (x₁,y₁) and (x₂,y₂ )

[tex]d = \sqrt {\left( {x_1 - x_2 } \right)^2 + \left( {y_1 - y_2 } \right)^2 }[/tex]

Hence "Midpoint is the average of x and y coordinates of the endpoint while the length of a line segment is given by the formula [tex]d = \sqrt {\left( {x_1 - x_2 } \right)^2 + \left( {y_1 - y_2 } \right)^2 }[/tex]".

For more about line segment,

https://brainly.com/question/25727583

#SPJ2

Write 52,640 in scientific notation. Please​

Answers

Answer:

5.264 * 10^4

Step-by-step explanation:

We put a decimal point to the right of the number, so

52640.

Then we move it to the left until we get a number that is between 1 and 10.

5.2640

This was four hops to the left.

We can remove the trailing 0, and write

5.264 * 10^4

10^4 because of the four hops

Answer:

5.264×10^4

Step-by-step explanation:

10^4 = 10,000 x 5.264 = 52,640

मच्ने costing Rs 10,000 is depreciated at 15% per annum find the value of maths in after 1 year? ​

Answers

Answer:

8500

Step-by-step explanation:

depreciation=10000×15%=1500

initial cost = 10000

final cost = initial cost- depreciation

=10000-1500

=8500

An alphabetic character representing a number

Answers

Answer:

a variable

Step-by-step explanation:

The answer is a variable because it's a letter that stands in place of a number

Answer:

simply refers to the type of Latin and Arabic characters representing the numbers 0 - 9, the letters A - Z

Simplify the expression. 3(7) – 3(3)

Answers

Answer:

[tex]12[/tex]

Step-by-step explanation:

[tex]3\times 7-3\times 3[/tex]

This term's numerical factors have been multiplied:-

[tex]=12[/tex]

OAmalOHopeO

Find the value of x.
95
38°
X

Answers

Answer:

47

Step-by-step explanation:

x is one of the three angles of a triangle. The other two are given as 95 and 38.

Together all three have to make 180 degrees.

x + 95 + 38 = 180

x + 133 = 180

x = 180 - 133

x = 47

Answer:

47

Step-by-step explanation:

as we know that sum of a triangle is always=180°

so x+38°+95°=180

so x+133=180

and x=180-133

so x= 47

The table shows the times for the 100-meter dash. What is the
difference between the men'stime and the women's time?
Write your answer in both decimal and fraction form.
Times for 100-meter Dash
Men's winner
9.58
2009
Women's winner
10.49
1988

Answers

Answer:

0.91 (make sure to include unit of measurement)

Step-by-step explanation:

10.49 - 9.58 = 0.91

What do these have in common??

Answers

Answer:

They can all be simplified to 2/3

Other Questions
Find the output, k, when the input, t, is -7.k= 10t - 19k= What was the Spanish king and queen's main goal in colonizing the New World ? Is 9pi rational or irrational write two things under the physical aspect of environment educationhelp me y more than 2 decreased by 8 what does erythropoietin do The lines shown below are perpendicular. Find the slope-intercept form of the equation of line y2. answer this correct and i will mark you brainliest 2 similar caps cost as much as 3 similar shirts.4 such caps and 3 such T-shirts cost $36.60Find the cost of a cap, and the cost of 3 T-Shirts Apple shows r_ _ _ _ _ _ to employees by treating them with fairness and kindness. By making inferences based on analysis of a character, the reader is able to determine a character'sO actionsO developmentO themeO motivationNextSubmitSave and ExitMark this and return What is the least positive integer by which 1922 should be divided so that the resultant integer is aperfect square? Geography is more than just facts about places. It's often the pointentire storyTrueFalse 1. Your friend needs quick energy! What kind of biomoleciste should he eat a lot of? 2. Why? Be specific, what do your cells do with this biomolecule that would help him run? 3. Name two kinds of foods have high amounts of this biomolecule. 5 increased by the product of 2 and t Select the correct answer. Consider this absolute value function. f(x)= |x-5| How can function f be written as a piecewise function?(multiple choice) Alexa has $300 in her bank account. Each week for 5 weeks she spends 18 dollars on things. How much money does she have after 5 weeks Guys answer this question Someone help please 1= 39 degrees 2: 40 degrees 3: 41 degrees 4: 42 degrees Who helped Hernn Corts and his soldiers defeat the Aztec?AtahualpaFrancisco Pizarrothe PortugueseAmerican Indian warriors